第八章に関するその他の例

  1. \(\sum n^{k}\{\sqrt{n + 1} - 2\sqrt{n} + \sqrt{n - 1}\}\) の収束性を議論せよ。\(k\) は実数とする。

    (Math. Trip. 1890.)

  2. 次の級数を考える: \[ \sum n^{r} \Delta^{k}(n^{s}) \] ここで \[ \Delta u_{n} = u_{n} - u_{n+1},\quad \Delta^{2} u_{n} = \Delta(\Delta u_{n}) \] などとする。この級数が収束するのは \(k \gt r + s + 1\) のときに限ると示せ。ただし \(s\) が \(k\) より小さい正の整数のときは級数が \(0\) となるので、この場合は除く。

    [第七章に関するその他の例 11 の結果から、\(\Delta^{k}(n^{s})\) が一般に \(n^{s-k}\) と同じオーダーにあることが分かる]

  3. 次を示せ: \[ \sum_{1}^{\infty} \frac{n^{2} + 9n + 5}{(n + 1)(2n + 3)(2n + 5)(n + 4)} = \frac{5}{36} \]

    (Math. Trip. 1912.)

    [一般項を部分分数で表す]

  4. 任意の \(n\) の有理関数 \(R(n)\) に対して、とある多項式 \(P(n)\) と定数 \(A\) に対する \(\sum \{R(n) - P(n) - (A/n)\}\) が収束すると示せ。特殊ケースとして \(R(n)\) が \(1/(an + b)\) および \((an^{2} + 2bn + c)/(\alpha n^{2} + 2\beta n + \gamma)\) である場合を考えよ。

  5. 級数 \[ 1 - \frac{1}{1 + z} + \frac{1}{2} - \frac{1}{2 + z} + \frac{1}{3} - \frac{1}{3 + z} + \cdots \] が収束すると示せ。ただし \(z\) が負の整数である場合は除く。

  6. 次の級数の収束と発散を調べよ: \[ \begin{gathered} \sum \sin\frac{a}{n},\quad \sum \frac{1}{n} \sin\frac{a}{n},\quad \sum (-1)^{n} \sin\frac{a}{n},\\ \sum \left(1 - \cos\frac{a}{n}\right),\quad \sum (-1)^{n} n\left(1 - \cos\frac{a}{n}\right) \end{gathered} \] \(a\) は実数とする。

  7. 次の級数の収束性を議論せよ: \[ \sum_{1}^{\infty} \left(1 + \frac{1}{2} + \frac{1}{3} + \cdots + \frac{1}{n}\right) \frac{\sin(n\theta + \alpha)}{n} \] \(\theta\) と \(\alpha\) は実数とする。

    (Math. Trip. 1989.)

  8. 級数 \[ 1 - \dfrac{1}{2} - \dfrac{1}{3} + \dfrac{1}{4} + \dfrac{1}{5} + \dfrac{1}{6} - \dfrac{1}{7} - \dfrac{1}{8} - \dfrac{1}{9} - \dfrac{1}{10} + \cdots \] が収束すると示せ。符号は \(1,\ 2,\ 3,\ 4,\ \ldots\) 項ずつ変化する。符号が \(1,\ 2,\ 4,\ 8,\ \ldots\) 項ずつ変化する級数は有限に振動することを示せ。

    (Math. Trip. 1908.)

  9. \(u_{1},\ u_{2},\ u_{3},\ \ldots\) が単調減少の正項列で極限が \(0\) のとき、 \[ \begin{gathered} u_{1} - \dfrac{1}{2}(u_{1} + u_{2}) + \dfrac{1}{3}(u_{1} + u_{2} + u_{3}) - \cdots, \\ u_{1} - \dfrac{1}{3}(u_{1} + u_{3}) + \dfrac{1}{5}(u_{1} + u_{3} + u_{5}) - \cdots \phantom{,} \end{gathered} \] は収束する。 [\((u_{1} + u_{2} + \cdots + u_{n})/n = v_{n}\) とすれば \(v_{1},\ v_{2},\ v_{3},\ \ldots\) も単調減少の正項列であり、極限は \(0\) となる (第四章のその他の例 8, 27)。ここから一つ目の級数が収束すると分かる。二つ目の級数については読者に任せる。特に級数 \[ \begin{gathered} 1 - \dfrac{1}{2}\left(1 + \dfrac{1}{2}\right) + \dfrac{1}{3}\left(1 + \dfrac{1}{2} + \dfrac{1}{3}\right) - \cdots,\\ 1 - \dfrac{1}{3}\left(1 + \dfrac{1}{3}\right) + \dfrac{1}{5}\left(1 + \dfrac{1}{3} + \dfrac{1}{5}\right) - \cdots\phantom{,} \end{gathered} \] は収束する]

  10. 各項が単調減少の正項級数 \(u_{0} + u_{1} + u_{2} + \cdots\) が発散するなら \[ \frac{u_{0} + u_{2} + \cdots + u_{2n}}{u_{1} + u_{3} + \cdots + u_{2n+1}} \to 1 \] が成り立つ。

  11. \(\alpha \gt 0\) なら \(\lim\limits_{p\to\infty} \sum\limits_{n=0}^{\infty} (p + n)^{-1-\alpha} = 0\) だと示せ。

  12. \(\lim\limits_{\alpha\to 0+} \alpha \sum\limits_{1}^{\infty} n^{-1-\alpha} = 1\) を示せ。 [§174 から \[ 0 \lt 1^{-1-\alpha} + 2^{-1-\alpha} + \cdots + (n - 1)^{-1-\alpha} - \int_{1}^{n} x^{-1-\alpha}\, dx \leq 1 \] が分かる。ここから \(\sum n^{-1-\alpha}\) が \(\dfrac{1}{\alpha}\) と \(\dfrac{1}{\alpha} + 1\) の間にあると示せる]

  13. 級数 \(\sum\limits_{1}^{\infty} u_{n}\) の和を求めよ。ここで \[ u_{n} = \frac{x^{n} - x^{-n-1}}{(x^{n} + x^{-n})(x^{n+1} + x^{-n-1}) } = \frac{1}{x - 1} \left(\frac{1}{x^{n} + x^{-n}} - \frac{1}{x^{n+1} + x^{-n-1}}\right) \] であり、級数は収束する \(x\) に対して定義されるとする。

    (Math. Trip. 1901.)

    [\(|x|\) が \(1\) より小さいとき級数の和は \(x/\{(x - 1)(x^{2} + 1)\}\) となる。\(x = 1\) なら \(u_{n} = 0\) だから和は \(0\) で、\(x = -1\) なら \(u_{n} = \frac{1}{2}(-1)^{n+1}\) だから級数は有限に振動する]

  14. 次の級数が収束するときの和を求めよ: \[ \frac{z}{1 + z} + \frac{2z^{2}}{1 + z^{2}} + \frac{4z^{4}}{1 + z^{4}} + \cdots,\quad \frac{z}{1 - z^{2}} + \frac{z^{2}}{1 - z^{4}} + \frac{z^{4}}{1 - z^{8}} + \cdots \] 指数は \(2\) のべきとする。

    [一つ目の級数は \(|z| \lt 1\) のときに限って収束し、和は \(z/(1 - z)\) となる。二つ目の収束は \(|z| \lt 1\) なら \(z/(1 - z)\) に収束し、\(|z| \gt 1\) なら \(1/(1 - z)\) に収束する]

  15. 全ての \(n\) に対して \(|a_{n}| \leq 1\) なら、方程式 \[ 0 = 1 + a_{1}z + a_{2}z^{2} + \cdots \] は絶対値が \(\frac{1}{2}\) より小さい根を持たない。唯一 \(a_{n} = -\operatorname{Cis}(n\theta)\) のときに限って根の絶対値が \(\frac{1}{2}\) となり、\(z = \frac{1}{2} \operatorname{Cis}(-\theta)\) が成り立つ。

  16. 再帰級数: べき級数 \(\sum a_{n}z^{n}\) が 再帰級数 (recurring series) であるとは、係数が次の関係を満たすことを言う: \[ a_{n} + p_{1}a_{n-1} + p_{2}a_{n-2} + \cdots + p_{k}a_{n-k} = 0 \qquad \text{(1)} \] ここで \(n \geq k\) であり、\(p_{1},\ p_{2},\ \ldots,\ p_{k}\) は \(n\) と独立とする。再帰級数は \(z\) の有理関数に展開できる。これを示すために、まず絶対値の十分小さい \(z\) に対して級数が収束することを見る。\(G\) を次の二つの実数の大きい方とする: \[ 1,\quad |p_{1}| + |p_{2}| + \cdots + |p_{k}| \] すると \(\text{(1)}\) から、\(n\) までの係数 \(a_{1},\ a_{2},\ \ldots a_{n}\) の絶対値の最大値を \(\alpha_{n}\) として \(|a_{n}| \leq G\alpha_{n}\) が分かる。よって \(n\) と無関係な \(K\) に対して \(|a_{n}| \lt KG^{n}\) が成り立つ。したがって考えている再帰級数は絶対値が \(1/G\) より小さい \(z\) で収束する。

    一方で級数 \(f(z) = \sum a_{n}z^{n}\) に \(p_{1}z,\ p_{2}z^{2},\ \ldots,\ p_{k}z^{k}\) を乗じて足すと、関係 \(\text{(1)}\) より第 \((k - 1)\) 項より後ろの項が全て消え、次の等式が得られる: \[ (1 + p_{1}z + p_{2}z^{2} + \cdots + p_{k}z^{k})f(z) = P_{0} + P_{1}z + \cdots + P_{k-1}z^{k-1} \] \(P_{0},\ P_{1},\ \ldots,\ P_{k-1}\) は定数である。多項式 \(1 + p_{1}z + p_{2}z^{2} + \cdots + p_{k}z^{k}\) を再帰級数の関係尺度 (scale of relation) と呼ぶ。

    有理関数は多項式と \(A/(z - a)^{p}\) という形の部分分数の和として表せるので、指数が負の整数のときの二項定理から、分母が \(z\) で割り切れない任意の有理関数は \(z\) の絶対値が小さいときに収束するべき級数に展開できると分かる。具体的には分母の根の絶対値の最小値 \(\rho\) に対して \(|z| \lt \rho\) なら収束する (参考: 第四章に関するその他の例 14など)。さらに一つ前の段落までの議論を逆回しにすれば、この条件を満たす級数が再帰級数であることも分かる。よってべき級数が再帰級数となる必要十分条件は、その級数が上述の条件を満たす \(\bm{z}\) の有理関数の展開となることである

  17. 差分方程式の解: 問題 16 の \(\text{(1)}\) のような関係式を \(\bm{a_{n}}\) に関する定数係数の線形差分方程式 (linear difference-equation in \(a_{n}\) with constant coefficients) と呼ぶ。この方程式の解法は例を示せば十分に説明できる。方程式 \[ a_{n} - a_{n-1} - 8a_{n-2} + 12a_{n-3} = 0 \] を考える。再帰べき級数 \(\sum a_{n}z^{n}\) を考えると、問題 16 と同様の方法でこの和が \[ \frac{a_{0} + (a_{1} - a_{0}) z + (a_{2} - a_{1} - 8a_{0}) z^{2}} {1 - z - 8z^{2} + 12z^{3}} = \frac{A_{1}}{1 - 2z} + \frac{A_{2}}{(1 - 2z)^{2}} + \frac{B}{1 + 3z} \] だと分かる。ここで \(A_{1},\ A_{2},\ B\) の \(a_{0},\ a_{1},\ a_{2}\) を使った表現は簡単に求まる。分数をそれぞれ個別に展開すれば、\(z^{n}\) の係数を得る: \[ a_{n} = 2^{n}\{A_{1} + (n + 1) A_{2}\} + (-3)^{n} B \] \(A_{1},\ A_{2},\ B\) の値は最初の三つの係数 \(a_{0},\ a_{1},\ a_{2}\) に依存しており、もちろん \(a_{0},\ a_{1},\ a_{2}\) の値に制限はない。

  18. 差分方程式 \(u_{n} - 2\cos\theta u_{n-1} + u_{n-2} = 0\) の解は \(u_{n} = A\cos n\theta + B\sin n\theta\) である。\(A\) と \(B\) は任意定数を表す。

  19. \(u_{n}\) が \(n\) の \(k\) 次多項式なら \(\sum u_{n} z^{n}\) は再帰級数であり、関係尺度は \((1 - z)^{k+1}\) である。

    (Math. Trip. 1904.)

  20. \(\dfrac{9}{(z - 1)(z + 2)^{2}}\) を \(z\) の昇べきに展開せよ。

    (Math. Trip. 1913.)

  21. \(\dfrac{z}{1 + z + z^{2}}\) を \(z\) のべき級数に展開したときの \(z^{n}\) の係数を \(f(n)\) とする。このとき \[ \text{(1)}\ f(n) + f(n - 1) + f(n - 2) = 0,\quad \text{(2)}\ f(n) = \frac{\omega_{3}^{n} - \omega_{3}^{2n}}{\omega_{3} - \omega_{3}^{2}} \] を示せ。\(\omega_{3}\) は \(1\) の複素三重根とする。\(n\) が \(3k,\ 3k + 1,\ 3k + 2\) のとき \(f(n)\) が \(0,\ 1,\ -1\) だと示し、この等式から \(z/(1 + z + z^{2}) = z(1 - z)/(1 - z^{3})\) を導け。

  22. プレイヤーがコインを投げ、表が出たら二点、裏が出たら一点を獲得する。点数が \(n\) 以上になるまでコインを投げ続けるとき、総得点がちょうど \(n\) になる確率が \(\frac{1}{3}\{2 + (-\frac{1}{2})^{n}\}\) だと示せ。

    (Math. Trip. 1898.)

    [確率を \(p_{n}\) とすれば \(p_{n} = \frac{1}{2} (p_{n-1} + p_{n-2})\) が成り立つ。また \(p_{0} = 1,\ p_{1} = \frac{1}{2}\) である]

  23. 次を示せ: \[ \frac{1}{a + 1} + \frac{1}{a + 2} + \cdots + \frac{1}{a + n} = \binom{n}{1}\frac{1}{a + 1} - \binom{n}{2}\frac{1!}{(a + 1)(a + 2)} + \cdots \] \(n\) は正の整数で、\(a\) は \(-1,\ -2,\ \ldots,\ -n\) のいずれでもないとする。

    [右辺の項を部分分数に展開すれば示せる。\(a \gt -1\) なら、この結果は次の等式から簡単に得られる: \[ \int_{0}^{1} x^{a}\frac{1 - x^{n}}{1 - x}\, dx = \int_{0}^{1} (1 - x)^{a}\{1 - (1 - x)^{n}\}\frac{dx}{x} \] 証明するには \((1 - x^{n})/(1 - x)\) と \(1 - (1 - x)^{n}\) を \(x\) のべき級数で表し、各項を個別に積分する。この結果は代数的な恒等式であり、\(-1,\ -2,\ \ldots,\ -n\) を除いた全ての \(a\) で成り立つ]

  24. 級数の乗算を使って次を示せ: \[ \sum_{0}^{\infty} \frac{z^{n}}{n!} \sum_{1}^{\infty} \frac{(-1)^{n-1}z^{n}}{n·n!} = \sum_{1}^{\infty} \left(1 + \frac{1}{2} + \frac{1}{3} + \cdots + \frac{1}{n}\right) \frac{z^{n}}{n!} \]

    [\(z^{n}\) の係数は \[ \frac{1}{n!}\left\{ \binom{n}{1} - \frac{1}{2}\binom{n}{2} + \frac{1}{3}\binom{n}{3} - \cdots \right\} \] となるので、問題 23 で \(a = 0\) としたものを使う]

  25. \(n \to \infty\) で \(A_{n} \to A\) および \(B_{n} \to B\) なら \[ \frac{A_{1}B_{n} + A_{2}B_{n-1} + \cdots + A_{n}B_{1}}{n} \to AB \] が成り立つ。

    [\(A_{n} = A + \varepsilon_{n}\) とすると、この式は \[ A \frac{B_{1} + B_{2} + \cdots + B_{n}}{n} + \frac{\varepsilon_{1}B_{n} + \varepsilon_{2}B_{n-1} + \cdots + \varepsilon_{n}B_{1}}{n} \] に等しい。

    一つ目の項は \(AB\) に向かう (第四章に関するその他の例 27)。\(|B_{\nu}|\) の最大値より大きい適当な実数を \(\beta\) とすると、二つ目の項の絶対値は \(\beta\{|\varepsilon_{1}| + |\varepsilon_{2}| + \cdots + |\varepsilon_{n}|\}/n\) より小さい。よって二つ目の項は \(0\) に向かう]

  26. \(c_{n} = a_{1}b_{n} + a_{2}b_{n-1} + \cdots + a_{n}b_{1}\) で \[ \begin{gathered} A_{n} = a_{1} + a_{2} + \cdots + a_{n},\\ B_{n} = b_{1} + b_{2} + \cdots + b_{n},\\ C_{n} = c_{1} + c_{2} + \cdots + c_{n}\hphantom{,} \end{gathered} \] なら \[ C_{n} = a_{1}B_{n} + a_{2}B_{n-1} + \cdots + a_{n}B_{1} = b_{1}A_{n} + b_{2}A_{n-1} + \cdots + b_{n}A_{1} \] および \[ C_{1} + C_{2} + \cdots + C_{n} = A_{1}B_{n} + A_{2}B_{n-1} + \cdots + A_{n}B_{1} \] だと示せ。

    この結果を使って、\(\sum a_{n}\) と \(\sum b_{n}\) が収束して和が \(A\) と \(B\) のとき、つまり \(A_{n} \to A,\ \) \(B_{n} \to B\) のとき \[ \frac{C_{1} + C_{2} + \cdots + C_{n}}{n} \to AB \] だと示せ。さらに \(\bm{c_{n}}\) が収束するなら和は \(\bm{AB}\) であることを導け。この結果は級数の積に関するアーベルの定理として知られる。級数 \(\sum a_{n}\) と \(\sum b_{n}\) が絶対収束するならこの方法で積が計算できることは前に見たが、アーベルの定理によると一方あるいは両方が絶対収束せずとも、積の級数が収束することさえ分かれば、この方法で積が計算できる。

  27. 次を示せ: \[ \begin{aligned} \dfrac{1}{2} \left(1 - \dfrac{1}{2} + \dfrac{1}{3} - \cdots\right)^{2} & = \dfrac{1}{2} - \dfrac{1}{3} \left(1 + \dfrac{1}{2}\right) + \dfrac{1}{4} \left(1 + \dfrac{1}{2} + \dfrac{1}{3}\right) - \cdots,\\ \dfrac{1}{2} \left(1 - \dfrac{1}{3} + \dfrac{1}{5} - \cdots\right)^{2} & = \dfrac{1}{2} - \dfrac{1}{4} \left(1 + \dfrac{1}{3}\right) + \dfrac{1}{6} \left(1 + \dfrac{1}{3} + \dfrac{1}{5}\right) - \cdots \end{aligned} \]

    [級数の収束性を示すには問題 9 を使う]

  28. 積分 \(\displaystyle\int_{0}^{\pi} \sin^{m} x (1 - \cos x)^{n}\, dx\) が収束するのは \(m\) と \(n\) がどんな値のときか? [\(m + 1\) と \(m + 2n + 1\) が正のとき]

  29. \(a \gt 1\) で次の等式が成り立つと示せ: \[ \int_{-1}^{1} \frac{dx}{(a - x) \sqrt{1 - x^{2}}} = \frac{\pi}{\sqrt{a^{2} - 1}} \]

  30. 次の等式を示せ: \[ \begin{alignedat}{2} \int_{0}^{\infty} F\{\sqrt{x^{2} + 1} + x\}\, dx & = \dfrac{1}{2}\int_{1}^{\infty} & & \left(1 + \frac{1}{y^{2}}\right) F(y) \, dy,\\ \int_{0}^{\infty} F\{\sqrt{x^{2} + 1} - x\}\, dx & = \dfrac{1}{2}\int_{0}^{1} & & \left(1 + \frac{1}{y^{2}}\right) F(y)\, dy \end{alignedat} \] 特に \(n \gt 1\) なら次の等式が成り立つと示せ: \[ \int_{0}^{\infty} \frac{dx}{\{\sqrt{x^{2} + 1} + x\}^{n}} = \int_{0}^{\infty} \{\sqrt{x^{2} + 1} - x\}^{n}\, dx = \frac{n}{n^{2} - 1} \]

    [この例およびこれからの例では、積分が §177 からの議論で考えた意味を持つような関数だけが積分記号の中に生じるとする]

  31. \(a\) と \(b\) が正の実数で \(2y = ax - (b/x)\) なら、\(x\) が \(0\) から \(\infty\) まで増加するとき \(y\) が \(-\infty\) から \(\infty\) に増加すると示せ。これを使って \[ \begin{aligned} \int_{0}^{\infty} f\left\{\dfrac{1}{2}\left(ax + \frac{b}{x}\right)\right\} dx & = \frac{1}{a} \int_{-\infty}^{\infty} f\{\sqrt{y^{2} + ab}\} \left\{1 + \frac{y}{\sqrt{y^{2} + ab}}\right\} dy\\ & = \frac{2}{a} \int_{0}^{\infty} f\{\sqrt{y^{2} + ab}\}\, dy \end{aligned} \] を示せ。

  32. \(a\) と \(b\) が正の実数で \(2y = ax + (b/x)\) なら、\(\sqrt{ab}\) より大きい任意の \(y\) には二つの \(x\) が対応すると示せ。二つの \(x\) の大きい方を \(x_{1}\) として小さい方を \(x_{2}\) とすると、\(y\) が \(\sqrt{ab}\) から \(\infty\) に増加するとき \(x_{1}\) は \(\sqrt{b/a}\) から \(\infty\) に増加し、\(x_{2}\) は \(\sqrt{b/a}\) から \(0\) に減少すると示せ。さらに \[ \begin{aligned} \int_{\sqrt{b/a}}^{\infty} f(y)\, dx_{1} & = \frac{1}{a} \int_{\sqrt{ab}}^{\infty} f(y) \left\{\frac{y}{\sqrt{y^{2} - ab}} + 1\right\} dy,\\ \int_{0}^{\sqrt{b/a}} f(y)\, dx_{2} & = \frac{1}{a} \int_{\sqrt{ab}}^{\infty} f(y) \left\{\frac{y}{\sqrt{y^{2} - ab}} - 1\right\} dy \end{aligned} \] および \[ \int_{0}^{\infty} f\left\{\dfrac{1}{2}\left(ax + \frac{b}{x}\right)\right\} dx = \frac{2}{a} \int_{\sqrt{ab}}^{\infty} \frac{yf(y)}{\sqrt{y^{2} - ab}}\, dy = \frac{2}{a} \int_{0}^{\infty} f\{\sqrt{z^{2} + ab}\}\, dz \] を導け。

  33. 次の等式を示せ: \[ \int_{0}^{\pi} f(\sec\dfrac{1}{2}x + \tan\dfrac{1}{2}x)\frac{dx}{\sqrt{\sin x}} = \int_{0}^{\pi} f(\cosec x)\frac{dx}{\sqrt{\sin x}} \]

  34. \(a\) と \(b\) が正なら \[ \int_{0}^{\infty} \frac{dx}{(x^{2} + a^{2})(x^{2} + b^{2})} = \frac{\pi}{2ab(a + b)},\quad \int_{0}^{\infty} \frac{x^{2}\, dx}{(x^{2} + a^{2})(x^{2} + b^{2})} = \frac{\pi}{2(a + b)} \] が成り立つ。これを使って、\(\alpha,\ \beta,\ \gamma\) が正で \(\beta^{2} \geq \alpha\gamma\) なら \[ \int_{0}^{\infty} \frac{dx}{\alpha x^{4} + 2\beta x^{2} + \gamma} = \frac{\pi}{2\sqrt{2\gamma A}}, \quad \int_{0}^{\infty} \frac{x^{2}\, dx}{\alpha x^{4} + 2\beta x^{2} + \gamma} = \frac{\pi}{2\sqrt{2\alpha A}} \] で \(A = \beta + \sqrt{\alpha\gamma}\) だと示せ。さらに \(f(y) = 1/(c^{2} + y^{2})\) として問題 31 の結果を導け。最後の二つの結果は \(\beta^{2} \lt \alpha\gamma\) でも正しいが、その場合の証明は難しい。

  35. \(b\) が正なら \[ \int_{0}^{\infty} \frac{x^{2}\, dx}{(x^{2} - a^{2})^{2} + b^{2}x^{2}} = \frac{\pi}{2b},\quad \int_{0}^{\infty} \frac{x^{4}\, dx}{\{(x^{2} - a^{2})^{2} + b^{2}x^{2}\}^{2}} = \frac{\pi}{4b^{3}} \] だと示せ。

  36. シュワルツの不等式 (第七章に関するその他の例 42) を第一種および第二種の無限積分に拡張せよ。

  37. §178 の終わりで考えた関数を \(\phi(x)\) とする。このとき \[ \int_{0}^{\infty} \phi(x)\, dx = \sum_{0}^{\infty} \frac{1}{(n + 1)^{2}} \] を示せ。

  38. 次を示せ: \[ \begin{alignedat}{2} \int_{1}^{\infty} dx \left(\int_{1}^{\infty} \frac{x - y}{(x + y)^{3}}\, dy\right) & = -1, \quad & \int_{1}^{\infty} dy \left(\int_{1}^{\infty} \frac{x - y}{(x + y)^{3}}\, dx\right) & = 1;\\ \int_{1}^{\infty} dx \left(\int_{1}^{\infty} \frac{x^{2} - y^{2}}{(x^{2} + y^{2})^{2}}\, dy\right) & = -\dfrac{1}{4}\pi, \quad & \int_{1}^{\infty} dy \left(\int_{1}^{\infty} \frac{x^{2} - y^{2}}{(x^{2} + y^{2})^{2}}\, dx\right) & = \dfrac{1}{4}\pi \end{alignedat} \]

    積分区間が \(0\) から \(1\) のときに対する同様の結果を示せ。

    (Math. Trip. 1913.)

広告